12 svar
202 visningar
L1vL är nöjd med hjälpen
L1vL 315 – Fd. Medlem
Postad: 19 maj 2021 23:49 Redigerad: 19 maj 2021 23:52

2018 Q20, ersättningsresistans

Hej, 

Hur ska man tänka kring det som är inringat i orange? Visste inte ifall jag skulle tolka det som parallell-, eller seriekoppling. Kan någon se i vilket steg jag gör fel? 

Pieter Kuiper Online 7040
Postad: 19 maj 2021 23:56

Du ska inse att det inte går någon ström genom motståndet i mitten. Du kan ta bort den.

L1vL 315 – Fd. Medlem
Postad: 20 maj 2021 00:15

Det läser jag med från facit, ingen spänning ingen ström. Det jag inte förstår är vilka symmetrier jag ska titta efter eller var det är serie-, respektive parallellkoppling. Det borde vara en nod i toppen, dvs att resistorerna över horisontal-linjen är i parallell. Men, jag tror inte att man kan säga att de är i parallell med varandra. Pga av förvirringen valde jag att slå ihop de tre resistorerna i minus x-led till en, men det blev fel, vilket är den tredje saken jag inte förstår.  

Pieter Kuiper Online 7040
Postad: 20 maj 2021 00:24
L1vL skrev:

Det läser jag med från facit, ingen spänning ingen ström. Det jag inte förstår är vilka symmetrier jag ska titta efter eller var det är serie-, respektive parallellkoppling.  

Detta går inte att lösa med serie- och parallellkoppling.

Är inte symmetrin uppenbar? Alla motstånd är lika stora.

Du hade alltså redan facit som förklarar varför det inte går någon ström genom motståndet i mitten: ingen spänning.

 

MrPillow01 14
Postad: 20 maj 2021 00:29
Pieter Kuiper skrev:
L1vL skrev:

Det läser jag med från facit, ingen spänning ingen ström. Det jag inte förstår är vilka symmetrier jag ska titta efter eller var det är serie-, respektive parallellkoppling.  

Detta går inte att lösa med serie- och parallellkoppling.

Är inte symmetrin uppenbar? Alla motstånd är lika stora.

Du hade alltså redan facit som förklarar varför det inte går någon ström genom motståndet i mitten: ingen spänning.

 

Varför finns ingen spänning över den vertikala? Vilken metod kan man använda för att komma fram till det? Tänker på potentialvandring, men det verkar som att du kan en enklare regel.

Pieter Kuiper Online 7040
Postad: 20 maj 2021 00:32

Tänk vad potentialerna är om man tar bort det där motståndet.

L1vL 315 – Fd. Medlem
Postad: 20 maj 2021 00:49 Redigerad: 20 maj 2021 00:49

Den symmetrin jag kan se är att det går att flippa positiva y-led över negativa, syftar man på det? Blir fel när jag räknar på det viset med 

Pieter Kuiper Online 7040
Postad: 20 maj 2021 00:56 Redigerad: 20 maj 2021 00:58
L1vL skrev:

Den symmetrin jag kan se är att det går att flippa positiva y-led över negativa, syftar man på det?  

Jag fattar inte.

Men om nu potentialen i A skulle var 5 volt och potentialen i B noll, hur stora är då potentialerna i de andra två noderna? (Lättare om du först tänker bort motståndet mellan dessa noder.)

L1vL 315 – Fd. Medlem
Postad: 20 maj 2021 00:57

Den här metoden skulle jag vilja använda (hittepå-krets, vet inte ifall slutresistansen är korrekt): 

Går inte det med den här uppgiften? 

L1vL 315 – Fd. Medlem
Postad: 20 maj 2021 01:03
Pieter Kuiper skrev:
L1vL skrev:

Den symmetrin jag kan se är att det går att flippa positiva y-led över negativa, syftar man på det?  

Jag fattar inte.

Men om nu potentialen i A skulle var 5 volt och potentialen i B noll, hur stora är då potentialerna i de andra två noderna? (Lättare om du först tänker bort motståndet mellan dessa noder.)

Jag vet inte :( 

L1vL 315 – Fd. Medlem
Postad: 20 maj 2021 01:06
Pieter Kuiper skrev:
L1vL skrev:

Den symmetrin jag kan se är att det går att flippa positiva y-led över negativa, syftar man på det?  

Jag fattar inte.

Men om nu potentialen i A skulle var 5 volt och potentialen i B noll, hur stora är då potentialerna i de andra två noderna? (Lättare om du först tänker bort motståndet mellan dessa noder.)

Tänkte att man kunde flippa över en triangel över den andra, likt detta:

 

MrPillow01 14
Postad: 20 maj 2021 01:48 Redigerad: 20 maj 2021 01:48

https://physics.stackexchange.com/questions/637648/why-does-the-vertical-resistor-have-no-voltage-across-it/637650?noredirect=1#comment1433482_637650 

Yngve Online 37871 – Livehjälpare
Postad: 20 maj 2021 08:02
MrPillow01 skrev: 

Varför finns ingen spänning över den vertikala? Vilken metod kan man använda för att komma fram till det? Tänker på potentialvandring, men det verkar som att du kan en enklare regel.

Du kan tänka så här:

  • Strömmen som går mellan A och B delar upp sig i de tre grenarna "övre", "mitten" och "undre".
  • Hur mycket ström som flyter i de tre grenarna avgörs av hur stor resistans respektive gren har.
  • Eftersom den övre grenen och den undre grenen ser identiska ut så har de samma resistans och därför flyter lika mycket ström genom den övre som genom den undre grenen.
  • Därmed kommer potentialen i den översta och den understa punkten att vara densamma.
Svara Avbryt
Close